Đến nội dung

sinh vien nội dung

Có 261 mục bởi sinh vien (Tìm giới hạn từ 20-04-2020)



Sắp theo                Sắp xếp  

#589040 Topic yêu cầu tài liệu toán cao cấp

Đã gửi bởi sinh vien on 15-09-2015 - 10:36 trong Tài liệu, chuyên đề Toán cao cấp

Cho mình hỏi ai có file cuốn " Exercises in Functional Analysis" của tác giả Dumitro Popa cho mình xin . Rất cảm ơn các bạn.




#561074 tuyển chọn các bài toán tính định thức

Đã gửi bởi sinh vien on 23-05-2015 - 09:12 trong Đại số tuyến tính, Hình học giải tích

Bài toán ( Sydney-2011) Cho m, n là hai số nguyên dương sao cho $m\geq n$ Gọi A là ma trậ vuông cấp n sao cho phần tử (i,j) bằng $C_{mj}^{i}$ . Tính det A .

    Đáp số : $detA=m^{\frac{n(n+1)}{2}}$




#560899 tuyển chọn các bài toán tính định thức

Đã gửi bởi sinh vien on 22-05-2015 - 14:15 trong Đại số tuyến tính, Hình học giải tích

Lời giải. Ta sẽ tính định thức bằng các xác định các giá trị riêng của ma trận.

  Trước tiên để cho thuận tiện trong các trình bày ta sẽ xét một trường hợp đặc biệt khi a=0 và bc=1

kí hiệu là $D_{n}$ . Đặt $P_{n}(\lambda )=det(\lambda I_{n}-D_{n})$ .

  Áp dụng khai triển Laplace liện tiếp cho ma trận $\lambda I_{n}-D_{n}$ , lần thứ nhất theo cột thứ nhất , lần thứ hai theo hàng thứ nhất, ta thu được hệ thức

$P_{n}(\lambda )=(-1)^{1+1}\lambda P_{n-1}(\lambda )+(-1)^{2+1}(-c) (-1)^{1+1}(-b)P_{n-2}(\lambda )=\lambda P_{n-1}(\lambda )-P_{n-2}(\lambda )$

  Ta qui ước $P_{0}(\lambda )=1$, bằng tính toán trực tiếp $P_{1}(\lambda )=1$ nên với mọi $n\geq 2$ ta luôn có

                     $P_{n}(\lambda )=\lambda P_{n-1}(\lambda )-P_{n-2}(\lambda )$

 Bằng phương pháp quy nạp ta dễ dàng nhận thấy;

                                 $P_{n}(2cos\theta )=\frac{sin(n+1)\theta )}{sin\theta }, 0< \theta < \pi$

  Dễ dàng nhận thấy $P_{n}(\lambda _{k})=0$ với $\lambda _{k}=2cos\frac{k\pi }{n+1},k=1,2,...n$  mà $degP_{n}(x)=n$ . Từ đây suy ra $\left \{ 2cos\frac{k\pi }{n+1} \right \}_{k=1}^{n}$ là tất cả các giá trị riêng của $D_{n}$.

   Trong trường hợp tổng quát ta thấy $A_{n}=aI_{n}+\sqrt{bc}\overline{D_{n}}$, trong đó $A_{n}$ là ma trận cho trong đề bài còn $\overline{D_{n}}$ có các phần tử trên đường chéo chính bằng 0 và thỏa mãn các tính chất trong trường hợp riêng mà ta đã khảo sát. Theo kết quả trên ta thấy $\left \{ a+2\sqrt{bc}cos\frac{k\pi }{n+1} \right \}_{k=1}^{n}$ là các giá trị riêng của $A_{n}$

   Do đó

       $det(A_{n})=\prod_{k=1}^{n}\left ( a+2\sqrt{bc}cos\frac{k\pi }{n+1} \right )=\prod_{k=1}^{n}\left ( a-2\sqrt{bc}cos\frac{(n+1-k)\pi }{n+1} \right )=\prod_{k=1}^{n}\left ( a-2\sqrt{bc}cos\frac{k\pi }{n+1} \right )$




#555380 tuyển chọn các bài toán tính định thức

Đã gửi bởi sinh vien on 21-04-2015 - 08:53 trong Đại số tuyến tính, Hình học giải tích

Sau đây là một số bài toán tính định thức theo các hướng độc đáo đã được sau trên các cuộc thi danh tiếng  (đề mục này sẽ được bổ sung dần dần )

   

Bài toán 1. (PUTNAM 2014)  Cho A là ma trận $n\times n$ trong đó phần tử ở hàng thứ i cột thứ j được cho bởi $\frac{1}{min(i,j)}$ với $1\leqslant i,j\leqslant n$ . Tính định thức của ma trận A

Lời giải. Từ điều kiện ở đầu ta thấy 

           A=$\begin{bmatrix} 1 & 1 &1 ...& 1& \\ 1& \frac{1}{2} &\frac{1}{2} ...& \frac{1}{2}& \\ 1 & \frac{1}{2}&\frac{1}{3} ...&\frac{1}{3} & \\1 &\frac{1}{2}&\frac{1}{3}...& \frac{1}{4}\\ ... & ... & ... &... \\ 1 & \frac{1}{2}&\frac{1}{3} ... & \frac{1}{n} & \end{bmatrix}$

   Để giải bài toán này ta sẽ áp dụng khai triển Laplace  cho hàng thứ n . Chú ý khi đó ma trận con thu được khi xóa hàng thứ n có dạng 

         \begin{bmatrix} 1 & 1& 1 & ... &1& 1\\1 & \frac{1}{2}&\frac{1}{2} & ...& \frac{1}{2} & \frac{1}{2}\\ ... & ... &... & ...&... & ...\\1 & \frac{1}{2}&\frac{1}{3} &..& \frac{1}{n-1} &\frac{1}{n-1} \end{bmatrix}$

Khi đó cột thứ n và cột thứ n-1 trùng nhau nên ma trận con chứa cả hai cột n và n-1 có định thức bằng 0 . Từ đó ta thấy

        $\begin{vmatrix} 1& 1 & 1 & ... &1 \\1 & \frac{1}{2} & \frac{1}{2} &... & \frac{1}{2}\\ 1& \frac{1}{2}&\frac{1}{2} &... &\frac{1}{2} \\ ...& ... &... & ... &... \\ 1& \frac{1}{2} &\frac{1}{3} & ... &\frac{1}{n} \end{vmatrix}=-\frac{1}{n-1}\begin{vmatrix} 1 & 1 & ... &1 \\ 1 & \frac{1}{2} &... &\frac{1}{2} \\... & ...& ... & ...\\ 1 & \frac{1}{2}&... &\frac{1}{n-1} \end{vmatrix}+\frac{1}{n}\begin{vmatrix} 1 & 1 & ... & 1\\1 & \frac{1}{2} & ... & \frac{1}{2}\\ ... &\... & ... &... \\ 1 & \frac{1}{2} &... &\frac{1}{n-1} \end{vmatrix}$

    Nếu đặt $D_{n}=det(A)$ trong đó A có cấp n thì $D_{n}=\left ( \frac{1}{n} -\frac{1}{n-1}\right ) D_{n}=\frac{-1}{n(n-1)}D_{n-1}$

    Sử dụng hệ thức truy hồi này và chú ý $D_{1}=1$ ta được

$D_{n}=\frac{-1}{n(n-1)}\frac{-1}{(n-1)(n-2)}...\frac{-1}{3.2}\frac{-1}{2.1}=\frac{(-1)^{n+1}}{n!(n-1)!}$

   Bài toán 2 (ĐHBK -2013) .Cho $x_{i},y_{i},1\leq i\leq n$ là các số phức với $x_{i}y_{j}$$\neq 1$ với mọi cặp $x_{i},y_{j}$.

  Tính định thức của ma trận $M=(m_{ij})_{n\times n}$ tróng đó $m_{ij}=\frac{1}{1-x_{i}y_{j}}$

lời giải. Để cho thuận tiện ta quy ước $D_{y_{1}y_{2}...y_{n}}^{x_{1}x_{2}...x_{n}}=det(M)$. Ta thấy

 n=2 : $D_{y_{1}y_{2}}^{x_{1}x_{2}}=\begin{vmatrix} \frac{1}{1-x_{1}y_{1}} &\frac{1}{1-x_{1}y_{2}} \\ \frac{1}{1-x_{2}y_{1}} &\frac{1}{1-x_{2}y_{2}} \end{vmatrix}=\frac{(x_{1}-x_{2})(y_{1}-y_{2})}{(1-x_{1}y_{1})(1-x_{1}y_{2})(1-x_{2}y_{1})(1-x_{2}y_{2})}$  ( tính toán tương đối đơn giản nên mình không nêu ra cụ thể )

Ta có thể tính trực tiếp thêm một số giá trị của n . Dự đoán :

$D_{y_{1}...y_{n}}^{x_{1}...x_{n}}=\frac{\prod_{1\leqslant i< j\leqslant n}(x_{i}-x_{j})(y_{i}-y_{j})}{\prod_{1\leq i,j\leq n}(1-x_{i}y_{j})}$. Ta sẽ chứng minh quy nạp công thức này.

   Áp dụng khai triển Laplace cho cột thứ nhất ta được

$D_{y_{1}..y_{n+1}}^{x_{1}...y_{n+1}}=\sum_{i=1}^{n+1}(-1)^{i+1}\frac{1}{1-x_{i}y_{1}}D_{y_{2}...y_{n+1}}^{x_{1}...x_{i-1}x_{i+1}...x_{n+1}}$

$=\sum_{i=1}^{n+1}(-1)^{i-1}\frac{1}{1-x_{i}y_{1}}\frac{\prod_{1\leq k< l\leq n+1,k,l\neq i}(x_{k}-x_{l})\prod_{2\leq k< l\leq n+1}(y_{k}-y_{l})}{\prod_{1\leq k\leqslant n+1,k\neq i,2\leq l\leqslant n+1}(1-x_{k}y _{l})}$

$\sum_{i=1}^{n+1}(-1)^{i-1}\prod_{k=1,k\neq i}^{n+1}(1-x_{k}y_{1})\frac{\prod_{1\leqslant k< l\leq n+1}(x_{k}-x_{l})\prod_{k=2}^{n+1}(1-x_{i}y_{k})\prod_{2\leq k< l\leq n+1}(y_{k}-y_{l})}{\prod_{k=1}^{i-1}(x_{k}-x_{i})\prod_{k=i+1}^{n+1}(x_{i}-x_{k})\prod_{1\leqslant k,l\leqslant n+1}(1-x_{k}y_{l})}$

$=\left ( \sum_{i=1}^{n+1}\prod_{k=2}^{n+1}(1-x_{i}y_{k})\prod_{k=1,k\neq i}^{n+1} \frac{1-x_{k}y_{1}}{x_{i}-x_{k}}\right )\frac{\prod_{1\leq k< l\leq n+1}(x_{k}-x_{l})\prod_{2\leq k< l\leq n+1}(y_{k}-y_{l})}{\prod_{1\leqslant k,l\leq n+1}(1-x_{k}y_{l})}$

 Đặt $P(x)=\prod_{k=2}^{n+1}(1-xy_{k})$ thì $P(x)$ là một đa thức bậc n và $P(x_{i})=\prod_{k=2}^{n+1}(1-x_{i}y_{k}),i=1,2,..n+1$ . Áp dụng công thức Lagrange ta được:

  $P(x)=\sum_{i=1}^{n+1}\prod_{k=2}^{n+1}(1-x_{i}y_{k})\prod_{k=1,k\neq i}^{n+1}\frac{x-x_{k}}{x_{i}-x_{k}}$

 Thay $x=\frac{1}{y_{1}}$ , nhân hai vế cho $y_{1}^{n}$ ta được

      $\prod_{k=2}^{n+1}(y_{1}-y_{k})=\sum_{i=1}^{n+1}\prod_{k=2}^{n+1}(1-x_{i}y_{k})\prod_{k=1,k\neq i}^{n+1}\frac{1-x_{k}y_{1}}{x_{i}-x_{k}}$.

  Từ đây ta dễ dàng thu được kết quả mong muốn.




#566709 tuyển chọn các bài toán tính định thức

Đã gửi bởi sinh vien on 18-06-2015 - 18:45 trong Đại số tuyến tính, Hình học giải tích

Tiếp theo chúng ta sẽ ôn tập lại phương pháp giá trị riêng thông qua một ví dụ nhỏ.

Bài toán ( Saint Peterburg -2007) Cho ma trận $M=(m_{ij})_{n\times n},$ trong đó $m_{ij}=\begin{cases} a_{i}a_{j} & \text{ }i\neq j \\ a_{i}^{2}+k& \text{ if } i= j \end{cases}$.

Tính detM.

Lời giải. 

Đặt: $A=\begin{pmatrix} a_{1}^{2} &a_{1} a_{2} & ... & a_{1}a_{n}\\ a_{2}a_{a} &a_{2}^{2} &... & a_{2}a_{n}\\ ...& ... &... &... \\a_{n}a_{1} &a_{n}a_{2} & ... &a_{n}^{2} \\ & & & \end{pmatrix}$

thì khi đó detM=det(A+kE), trong đó E là ma trận đơn vị cấp n.

  Dễ dàng nhận thấy rankA=1 do đó 0 là một giá trị riêng của ma trận A và có số bội là n-1 nên giá trị riêng còn lại sẽ là $a_{1}^{2}+a_{2}^{2}+...+a_{n}^{2}$ nên ta suy ra đẳng thức bên dưới đây

              $det(A-\lambda E)=(-1)^{n}\lambda ^{n-1}(\lambda -(a_{1}^{2}+a_{2}^{2}+...+a_{n}^{2}))$.

Thay $\lambda =-k$ ta sẽ thu được đáp án cho câu hỏi ban đầu là $k^{n-1}(k+a_{1}^{2}+a_{2}^{2}+...+a_{n}^{2})$.

 Bên dưới đây là một file đề thi bằng tiếng Nga dành cho các bạn nghiên cứu thêm

 File gửi kèm  2007.pdf   172.63K   183 Số lần tải




#566808 tuyển chọn các bài toán tính định thức

Đã gửi bởi sinh vien on 19-06-2015 - 09:42 trong Đại số tuyến tính, Hình học giải tích

Bài toán (AMM11270) Gọi $S_{n}$ là ma trận vuông cấp n có các phần tử thuộc tập $\left \{ 1,2,...,n^{2} \right \}$ .Các phần tử được sắp xếp theo hình xoắn ốc theo chiều tăng của các giá trị.

Tính $detS_{n}$.

Đáp số: $detS_{n}=(-1)^{\frac{n(n-1)}{2}}4^{n-1}\frac{3n-1}{2}\prod_{k=0}^{n-2}\left ( k+\frac{1}{2} \right )$

File lời giải:

 File gửi kèm  AMM11270.pdf   63.73K   141 Số lần tải




#560858 tuyển chọn các bài toán tính định thức

Đã gửi bởi sinh vien on 22-05-2015 - 10:53 trong Đại số tuyến tính, Hình học giải tích

Bài toán ( Nordic-2011) Chứng minh rằng

$\begin{vmatrix} a & b & 0 & 0 & 0 & ... & 0 & 0 &0 \\ c &a &b & 0 & 0 & ... & 0 &0 &0 \\0 & c & a & b &0 & ... & 0 & 0 &0 \\... & ... & ... &... &... &... &... &... &... \\ 0 & 0 & 0 & 0 &0 & ... & a &b &0 \\0 & 0 & 0 & 0 & 0 & ... & c & a &b \\ 0 & 0 & 0 & 0 & 0 &... &0 &c & a \end{vmatrix}_{n\times n}=\prod_{k=1}^{n}\left ( a-2\sqrt{bc}cos\frac{k\pi }{n+1} \right )$




#562808 tuyển chọn các bài toán tính định thức

Đã gửi bởi sinh vien on 01-06-2015 - 10:05 trong Đại số tuyến tính, Hình học giải tích


Tiếp theo là một số bài toán tính toán định thức có trên tạp chí American Mathematical ....

Bài toán ( AMM-11179) Cho các số nguyên dương i, j đặt $m_{i,j}=\left\{\begin{matrix} -1 &i+1\equiv 0(mod j) \\0 & i+1\not\equiv 0(modj) \end{matrix}\right.$.Gỉa sử $M_{n}$ là ma trận vuông cấp n-1 có phần tử (i,j) là $m_{i,j}$.

  Chứng minh rằng $det(M_{n})=\mu (n)$, trong đó $\mu$ là hàm Mobius . ( Tạp chí Epsilon)




#561548 tuyển chọn các bài toán tính định thức

Đã gửi bởi sinh vien on 25-05-2015 - 18:59 trong Đại số tuyến tính, Hình học giải tích

Bài toán ( ĐH-FPT 2013 ) Tính định thức sau

         $\begin{vmatrix} x+a_{1} &a_{2} & ... &a_{n} \\a_{1} & x+a_{2} & ... & a_{n}\\ ... &... &... &... \\a_{1} & a_{2} &... & x+a_{n} \end{vmatrix}$

Cách 1 ( Biến đổi sơ  cấp )  Cộng  tất cả các cột 2 ,3,...,n vào cột đầu tiên ta thu được

                                 $\begin{vmatrix} x+a_{1}+a_{2}+...+a_{n} & a_{2} &... & a_{n}\\ x+a_{1}+a_{2}+...+a_{n}&x+a_{2} &... &a_{n} \\ ...& ....& ... &.... \\x+a_{1}+a_{2}+...+a_{n} &a_{2} &... &x+a_{n} \end{vmatrix}$

 Tiếp theo trừ hàng thứ 1 cho các hàng 2,3,...n ta được

                  $\begin{vmatrix} x+a_{1}+a_{2}+...+a_{n} & a_{2} & ... & a_{n}\\ 0 & x & ... & a_{n}\\... & ... & ... & ...\\ 0 & 0 & ... & x \end{vmatrix}$

  Đến đây dễ  thấy giá trị định thức cần tìm bằng $x^{n-1}(x+a_{1}+a_{2}+...+a_{n})$

Cách 2 ( Lý thuyết về giá trị riêng - đa thức đặc trưng )  Đặt  $P(\lambda )=det(A-\lambda I_{n})$,

trong đó $A=\begin{bmatrix} a_{1} &a_{2} & ... &a_{n} \\ a_{1} & a_{2} & ... &a_{n} \\ ...& ... & ... &... \\a_{1} & a_{2}& ... & a_{n} \end{bmatrix}$. 

  Ta thấy rank A=1 , nên 0 là một  giá trị riêng của ma trận A và có số bội bằng $n-rankA=n-1$. 

  Theo đó giá trị riêng còn lại của ma trận A  bằng $a_{1}+a_{2}+...+a_{n}$ . Lưu ý là hệ  số của $\lambda ^{n}$ trong $P(\lambda )$ là $(-1)^{n}$

nên $P(\lambda )=det(A-\lambda I_{n})(-1)^{n}\lambda ^{n-1}(\lambda +a_{1}+a_{2}+...+a_{n})$ 

 Thay $\lambda =-x$ thay thấy $det(A+xI_{n})=x^{n-1}(x+a_{1}+a_{2}+...+a_{n})$.

  Chú ý vế trái là định thức ta cần tính.

      




#558175 tuyển chọn các bài toán tính định thức

Đã gửi bởi sinh vien on 07-05-2015 - 12:35 trong Đại số tuyến tính, Hình học giải tích

Bài toán:(Putnam 2009 )Cho n > 3, n nguên dương. Tính định thức:

 $D_{n}=\begin{vmatrix} cos1 &cos2 &... &cosn \\ cos(n+1) & cos(n+2) &... &cos2n \\... &... &... &... \\cos(n^{2}-n+1) & cos(n^{2}-n+2) &... &cosn^{2} \end{vmatrix}$

Lời giải.  Lấy cột thứ 3 + cột thứ 1$\rightarrow$ cột thứ 1 và sử dụng biến đổi lượng giác , ta được :

$D_{n}=\begin{vmatrix} 2cos2cos1 &cos2 &... & ... &... cos(n) \\2cos(n+2)cos1 &cos(n+2) &... & ... &cos2n \\... & ... & & ... &... \\... &... & ... & ... &... \\2cos(n^{2}-n+2)cos1 & cos(n^{2}-n+2) &... &... & cosn^{2} \end{vmatrix}$

   Lúc này cột thứ 1= 2cos1 cột thứ 2 $\Rightarrow$ det$D_{n}=0$.

Bài toán . Xét các số phức$z_{1},z_{2},...,z_{2n}$ thỏa mãn điều kiện  $\left | z_{1} \right |=\left | z_{2} \right |=...=\left | z_{n+3} \right |$ và $argz_{1}\geqslant argz_{2}\geqslant ...\geq argz_{n+3}$.

  Tính định thức của ma trận B ,trong đó  $b_{ij}=\left | z_{i}-z_{j+n} \right |,i,j\in \left \{ 1,2,...n \right \}$.

 Lời giải . Ta có nhận xét sau bài toán đề cập đến modun và argument của một số phức nên ta sẽ sử dụng dạng lượng giác của số phức trong các tính toán. 

    Với hai số phức $z=r(cosx+isinx);\omega =r(cosy+isiny)\Rightarrow \left | z-\omega \right |=2r\left | sin\left ( \frac{x-y}{2} \right ) \right |$. 

  Kết hợp với điều kiện ở đầu bài, ta thấy B có dạng như sau:

$B=(2r)^{n}\begin{vmatrix} sin(x_{1}-x_{n+1}) &sin(x_{1}-x_{n+2}) & sin(x_{1}-x_{n+3}) & ...\\... &... & ... &... \\ sin(x_{n}-x_{n+1}) & sin\left ( x_{n}-x_{n+2} \right ) & sin(x_{n}-x_{n+3}) & ... \end{vmatrix}$

 trong đó $x_{i}$ kí hiệu tương ứng argument của $z_{i}$ và r=$\left | z_{1} \right |=\left | z_{2} \right |=...=\left | z_{2n} \right |$

 Nếu ta tách det B theo các cột thì dễ dàng nhận thấy det B=0 

 

 




#558174 tuyển chọn các bài toán tính định thức

Đã gửi bởi sinh vien on 07-05-2015 - 12:35 trong Đại số tuyến tính, Hình học giải tích

Bài toán:(Putnam 2009 )Cho n > 3, n nguên dương. Tính định thức:

 $D_{n}=\begin{vmatrix} cos1 &cos2 &... &cosn \\ cos(n+1) & cos(n+2) &... &cos2n \\... &... &... &... \\cos(n^{2}-n+1) & cos(n^{2}-n+2) &... &cosn^{2} \end{vmatrix}$

Lời giải.  Lấy cột thứ 3 + cột thứ 1$\rightarrow$ cột thứ 1 và sử dụng biến đổi lượng giác , ta được :

$D_{n}=\begin{vmatrix} 2cos2cos1 &cos2 &... & ... &... cos(n) \\2cos(n+2)cos1 &cos(n+2) &... & ... &cos2n \\... & ... & & ... &... \\... &... & ... & ... &... \\2cos(n^{2}-n+2)cos1 & cos(n^{2}-n+2) &... &... & cosn^{2} \end{vmatrix}$

   Lúc này cột thứ 1= 2cos1 cột thứ 2 $\Rightarrow$ det$D_{n}=0$.

Bài toán . Xét các số phức$z_{1},z_{2},...,z_{2n}$ thỏa mãn điều kiện  $\left | z_{1} \right |=\left | z_{2} \right |=...=\left | z_{n+3} \right |$ và $argz_{1}\geqslant argz_{2}\geqslant ...\geq argz_{n+3}$.

  Tính định thức của ma trận B ,trong đó  $b_{ij}=\left | z_{i}-z_{j+n} \right |,i,j\in \left \{ 1,2,...n \right \}$.

 Lời giải . Ta có nhận xét sau bài toán đề cập đến modun và argument của một số phức nên ta sẽ sử dụng dạng lượng giác của số phức trong các tính toán. 

    Với hai số phức $z=r(cosx+isinx);\omega =r(cosy+isiny)\Rightarrow \left | z-\omega \right |=2r\left | sin\left ( \frac{x-y}{2} \right ) \right |$. 

  Kết hợp với điều kiện ở đầu bài, ta thấy B có dạng như sau:

$B=(2r)^{n}\begin{vmatrix} sin(x_{1}-x_{n+1}) &sin(x_{1}-x_{n+2}) & sin(x_{1}-x_{n+3}) & ...\\... &... & ... &... \\ sin(x_{n}-x_{n+1}) & sin\left ( x_{n}-x_{n+2} \right ) & sin(x_{n}-x_{n+3}) & ... \end{vmatrix}$

 trong đó $x_{i}$ kí hiệu tương ứng argument của $z_{i}$ và r=$\left | z_{1} \right |=\left | z_{2} \right |=...=\left | z_{2n} \right |$

 Nếu ta tách det B theo các cột thì dễ dàng nhận thấy det B=0 

 

 




#560844 tuyển chọn các bài toán tính định thức

Đã gửi bởi sinh vien on 22-05-2015 - 07:51 trong Đại số tuyến tính, Hình học giải tích

Bài toán ( Sydney-2005). Tính định thức của ma trận vuông A cấp n trong đó

                                                             $a_{ij}=\frac{(2i+2j-2)!}{2^{2i+2j-2}(i+j-1)!}$

Lời giải. Ta chứng minh kết quả sau:

  Cho  $\alpha \in \mathbb{R}$ , giả sử $C=(c_{ij})_{i,j=1}^{n}$ thỏa $c_{i,j+1}=(i+j-\alpha )c_{ij}$

 với mọi $1\leq i\leq n,1\leq j\leq n-1$ thì $detC=\prod_{i=1}^{n}(i-1)!c_{i,1}$




#559297 tuyển chọn các bài toán tính định thức

Đã gửi bởi sinh vien on 14-05-2015 - 11:14 trong Đại số tuyến tính, Hình học giải tích

  Lưu ý với các bạn bài toán này mình đã thay đổi yêu cầu để cho đơn giản bạn có thể tìm thấy nội dung đầy đủ của nó ở Putnam 1984

 Bài toán . Cho n là một số nguyên dương. Gỉa sử a,b,x là các số thực , với $a^{2}\neq b^{2}$, Ma trận vuông $M_{n}$ cấp 2n trong đó phần tử (i,j) được xác định:

$m_{ij}=\left\{\begin{matrix} x & i=j\\ a & i\neq j &i+j =2k \\ b & i\neq j & i+j=2k+1 \end{matrix}\right.$ 

  Tính $detM_{n}$ theo a,b,n

Lời giải. Gọi N là ma trận $M_{n}$ khi x =a . Từ định nghĩa ta dễ dàng nhận thấy N có hạng bằng 2 ( chỉ có hai hàng phân biệt ). Do đó 0 là một giá trị riêng của N với số bội bằng 2n-2.

  Nếu gọi e là ma trận $2n\times 1$ với các phần tử bằng một . 

  Với một chút tính toán ta sẽ thấy Ne=n(a+b)e- điều này có nghĩa là n(a+b) là một giá trị riêng khác của N ứng với vector riêng e

   Ta thấy vết của N bằng 2an $\Rightarrow$ Tồn tại một giá trị riêng thứ ba bằng 2an-n(a+b)=n(a-b)

 Từ đây dễ dàng suy ra rằng

$det(N-\lambda I_{2n})=\lambda ^{2n-2}(\lambda -n(a+b))(\lambda -n(a-b))$

  Để giải quyết trọn vẹn bài toán ta chú ý một điểm quan trọng sau $M_{n}=N-(a-x)I_{2n}$

  Thay $\lambda =a-x$ từ công thức trên ta suy ra

 $detM_{n}=(a-x)^{2n-2}(a-x-n(a-b))(a-x+n(a-b))$.

 




#560719 tuyển chọn các bài toán tính định thức

Đã gửi bởi sinh vien on 21-05-2015 - 16:37 trong Đại số tuyến tính, Hình học giải tích

Bài toán ( Sydney-2013). Tính định thức của ma trận vuông cấp n trong đó phần tử (i,j) bằng i nếu $i\neq j$ và bằng i+1 nếu i=j

Lời giải. Gọi $M_{n}$ là ma trận được định nghĩa, $I_{n}$ là ma trận đơn vị cấp n.

Khi đó ta thấy từ định nghĩa thì ma trận $M_{n}-I_{n}$ có hàng thứ i là (i,i,...,i) nên có hạng bằng 1 và định thức bằng 0 .

 Suy ra 1 là một giá trị riêng của ma trận $M_{n}$ có số bội bằng n-1.

Ta thấy vết của ma trận $M_{n}$ bằng $\sum_{i=1}^{n}(i+1)=\frac{n^{2}+3n}{2}$ nên giá trị riêng còn lại của ma trận $M_{n}$ bằng $\frac{n^{2}+3n}{2}-(n-1)=\frac{n^{2}+n+2}{2}$

  Do đó : $detM_{n}=1^{n-1}\times\frac{n^{2}+n+2}{2}=\frac{n^{2}+n+2}{2}$




#560145 tuyển chọn các bài toán tính định thức

Đã gửi bởi sinh vien on 18-05-2015 - 13:15 trong Đại số tuyến tính, Hình học giải tích

Bài toán. Cho ma trận vuông A cấp n , với $a_{ij}=(i,j)$, trong đó (i,j) là ước chung lớn nhất của i và j

Tính det A




#560738 tuyển chọn các bài toán tính định thức

Đã gửi bởi sinh vien on 21-05-2015 - 17:51 trong Đại số tuyến tính, Hình học giải tích

Bài toán ( Sydney-2009 ) Cho $A_{n}$ là ma trận vuông cấp n trong đó phần tử (i,j) bằng 1 nếu $n\leq i+j\leq n+1$ và bằng 0 trong các trường hợp còn lại.

 Tìm các giá trị riêng của $A_{n}$




#712253 Bài toán chéo hóa

Đã gửi bởi sinh vien on 09-07-2018 - 17:29 trong Tài liệu, chuyên đề Toán cao cấp

 dien

cho mình hỏi bạn có hiểu khái niệm đa thức đặc trưng của một tự đồng cấu là gì không?

đa thức đặc trưng của một tự đồng cấu là đa thức đặc trưng của ma trận biểu diễn của tự đồng cấu đó trên một cơ sở xác định




#713617 Bài toán chéo hóa

Đã gửi bởi sinh vien on 31-07-2018 - 23:15 trong Tài liệu, chuyên đề Toán cao cấp

Ok, bạn đã phát hiện dùm mình một lỗi đánh máy quan trọng.



#708512 Bài toán chéo hóa

Đã gửi bởi sinh vien on 16-05-2018 - 11:27 trong Tài liệu, chuyên đề Toán cao cấp

nếu bạn hiểu một đa thức là tách được khi tât cả các nghiệm đều là nghiệm đơn thì đã kể đến số bội của đa thức đó rồi chứ ! Mà đặc biệt là số bội bằng 1 hết nên thành ra số chiều cũng vậy. Mình sử dụng định nghĩa về tính tách được theo lối thứ hai: mọi nhân tử bất khả quy đều có nghiệm đơn, còn mình viết cụ thể ra chứ không xài lối diễn đạt bằng ngôn ngữ. Bởi vì dùng theo lối này ta có thể khảo sát bài toán trong phạm vi tổng quát hơn chứ !




#708410 Bài toán chéo hóa

Đã gửi bởi sinh vien on 15-05-2018 - 08:05 trong Tài liệu, chuyên đề Toán cao cấp

Điểm thứ nhất, thứ ba thì mình rất vui lòng lắng nghe sự đóng góp ý kiến tận tình của bạn, đặc biệt là điều thứ ba rất thú vị bởi vì nó là hướng tiếp cận trên ngôn ngữ thuần túy của không gian vector cho bài toán chéo hóa ma trận và khá kinh điển tìm đọc được ở nhiều tài liệu. Nhưng còn về điểm thứ nhì thì bạn có vẻ hơi hiểu sai về mặt thuật ngữ với vấn đề một chút, xin chỉnh lại tí xíu thôi. Vì mình xét trên một trường K tổng quát nên một đa thức có thể không phải lúc nào cũng tách được và khi tách được thì cũng không chắc gì là  số chiều của các không gian con riêng sẽ bằng với số bội của trị riêng tương ứng, cái này thì từng cái cụ thể mình mới có thể tính được số chiều của từng không gian con chỉ duy nhất khi các nghiệm đều có bội 1 thì hai thứ này luôn luôn trùng nhau mình có chỉ ra rồi. Tách được ở đây muốn chỉ rằng ta có thể phân tích đa thức đó thành tích các nhân tử bậc nhất đó bạn, Nói tóm lại là dùng tính tách thôi là ok trong vụ tam giác hóa rồi chứ cần quan tâm gì đến ba cái bội làm gì cho rắc rối, nếu mà có thêm được số chiều của không gian con riêng số bội của trị riêng tương ứng thì mới ok chéo hóa được. 




#708279 Bài toán chéo hóa

Đã gửi bởi sinh vien on 13-05-2018 - 20:08 trong Tài liệu, chuyên đề Toán cao cấp

File gửi kèm  Bài toán chéo hóa.pdf   464.85K   576 Số lần tải

File gửi kèm  Điều kiện cần và đủ để tam giác hóa một tự đồng cấu.pdf   217.8K   478 Số lần tải




#719629 Các chủ đề về cấu hình tổ hợp dành cho toán rời rạc

Đã gửi bởi sinh vien on 20-01-2019 - 09:09 trong Tài liệu, chuyên đề Toán cao cấp

File gửi kèm  Bất đẳng thức Fisher.pdf   257.89K   422 Số lần tải

[File gửi kèm  Khảo sát điều kiện tồn tại của một BIBD.pdf   346.97K   122 Số lần tải

File gửi kèm  Cấu hình tổ hợp liên kết với đồ thị strong regular.pdf   357.86K   150 Số lần tải




#562686 Các bài toán trên tạp chí American Mathematical Montly

Đã gửi bởi sinh vien on 31-05-2015 - 18:06 trong Tài nguyên Olympic toán

11783 File gửi kèm  AMM11783.pdf   95.65K   129 Số lần tải 

11782File gửi kèm  AMM11782 solution1 .pdf   110.96K   109 Số lần tải-File gửi kèm  AMM 11782 solution2.pdf   118.51K   135 Số lần tải

11781File gửi kèm  AMM11781.pdf   127.97K   142 Số lần tải




#562684 Các bài toán trên tạp chí American Mathematical Montly

Đã gửi bởi sinh vien on 31-05-2015 - 18:00 trong Tài nguyên Olympic toán

11793File gửi kèm  AMM11793.pdf   26.65K   106 Số lần tải

11792File gửi kèm  AMM11792.pdf   31.62K   113 Số lần tải

11791File gửi kèm  AMM11791 solution1.pdf   31.71K   118 Số lần tải-File gửi kèm  AMM 11791 solution2.pdf   113.76K   216 Số lần tải

11790File gửi kèm  AMM11790.pdf   29.75K   137 Số lần tải

11789File gửi kèm  AMM11789.pdf   32.02K   107 Số lần tải

11788File gửi kèm  AMM11788 solution1.pdf   96.55K   173 Số lần tải-File gửi kèm  AMM 11788 solution2.pdf   101.56K   188 Số lần tải

11787File gửi kèm  AMM11787.pdf   128.68K   120 Số lần tải




#559334 Một số bài toán cơ bản

Đã gửi bởi sinh vien on 14-05-2015 - 17:23 trong Tài liệu và chuyên đề Đại số đại cương

Bài toán (Putnam 1977) . Cho G là một nhóm , H là một nhóm con hữu hạn cấp h của G. Gỉa sử tồn tại phần tử $a\in G$ sao cho với mọi phần tử $x\in H$ ta luôn có $(xa)^{3}=1$

  Đặt  $P=\left \{ x_{1}ax_{2}a...x_{n}a,x_{i}\in H,i=\overline{1,n},n\in N^{*} \right \}$

  Chứng minh rằng $\left | P \right |\leq 3h^{2}$